LSAT and Law School Admissions Forum

Get expert LSAT preparation and law school admissions advice from PowerScore Test Preparation.

 Administrator
PowerScore Staff
  • PowerScore Staff
  • Posts: 8929
  • Joined: Feb 02, 2011
|
#24066
Complete Question Explanation

Flaw in the Reasoning. The correct answer choice is (C)

The flaw in this stimulus is found in the author’s assertion that, because two individual styles bring about two attributes, a combination of the two must cause both attributes to appear. This is certainly not true, especially with regard to aesthetics. Two styles may not mix well, and may not lead to either attribute’s being manifest in the hybrid style.

Answer choice (A): While this is a logical flaw, it is not one displayed in this stimulus, so this answer choice is incorrect.

Answer choice (B): The argument does not make this assertion, so this cannot be the flaw found here, and this answer choice cannot be correct.

Answer choice (C): This is the correct answer choice, which restates the point made in the discussion above: a combination of two styles may not bring about either intended attribute.

Answer choice (D): There is no need to make such a specification. Since this isn’t even a flaw, this cannot be the flaw manifest in this stimulus.

Answer choice (E): Like incorrect answer choice (D) above, this answer choice deals with irrelevant considerations. The argument does not require that we consider other architectural styles, so this is not a flaw, and this answer choice is incorrect.
User avatar
 LawSchoolDream
  • Posts: 57
  • Joined: Jan 18, 2024
|
#105351
can you pls expand on why not A? is it because the author DOES provide justification by calling modern arch dramatic, etc?
User avatar
 Chandler H
PowerScore Staff
  • PowerScore Staff
  • Posts: 105
  • Joined: Feb 09, 2024
|
#105567
LawSchoolDream wrote: Sun Feb 18, 2024 11:37 pm can you pls expand on why not A? is it because the author DOES provide justification by calling modern arch dramatic, etc?
Hi LawSchoolDream,

Not quite. The argument simply does not presume such a thing—it doesn't say that (for example) rounded arches and symmetry must be beautiful in order for the Roman architecture style to be beautiful. Since this presumption is not made in the question, (A) is irrelevant.

Does that make sense?
User avatar
 AnaSol
  • Posts: 23
  • Joined: Nov 20, 2023
|
#106488
Hi,

My prephrase was aligned with what C says. But I was between A and C and ended up choosing A.

Because A says:
presumes [-An architectural style that combines elements of both classical Roman and postmodern architecture-],
without providing justification,
that for an architectural style [the new style]
to have certain qualities, [beautiful and dramatic]
its components [Classical Roman architecture is beautiful + Postmodern architecture is dramatic]
must have those qualities [being beautiful and dramatic.]

What makes A incorrect?

Thanks!
User avatar
 Chandler H
PowerScore Staff
  • PowerScore Staff
  • Posts: 105
  • Joined: Feb 09, 2024
|
#106585
AnaSol wrote: Wed May 15, 2024 11:03 am Hi,

My prephrase was aligned with what C says. But I was between A and C and ended up choosing A.

Because A says:
presumes [-An architectural style that combines elements of both classical Roman and postmodern architecture-],
without providing justification,
that for an architectural style [the new style]
to have certain qualities, [beautiful and dramatic]
its components [Classical Roman architecture is beautiful + Postmodern architecture is dramatic]
must have those qualities [being beautiful and dramatic.]

What makes A incorrect?

Thanks!
Hi Ana,

Good question! The presumption that the author makes is that combining a beautiful style and a dramatic style will make a beautiful and dramatic style. However, answer choice (A) describes a scenario where the author presumes that a beautiful and dramatic style is made up of a beautiful style and a dramatic style.

The difference is what the author presumes. For example, I could say: "You're wearing a jacket and boots, so it must be raining today!" or "It's raining today, so you must be wearing a jacket and boots!" Both could be flawed assumptions, but they assume different things. Does that make sense?

Get the most out of your LSAT Prep Plus subscription.

Analyze and track your performance with our Testing and Analytics Package.